PSAT Reading Question 139: Answer and Explanation

Question: 139

Which choice provides the best evidence for the answer to the previous question?

  • A. Lines 9–12 ("Research scientists... possible")
  • B. Lines 21–24 ("The scientists... progress")
  • C. Lines 32–35 ("They removed... nose")
  • D. Lines 63–66 ("These exciting... fibers")

Correct Answer: B

Explanation:

B

Difficulty: Medium

Category: Command of Evidence

Getting to the Answer: Think about how you selected the correct answer for the previous question. Use that information to help you choose the correct answer to this question. In paragraph 2, the author explains that the patient selected for the experimental treatment had not regained mobility despite intensive physical therapy. This provides the strongest support for the answer to the previous question, so (B) is correct.

Test Information

  • Use your browser's back button to return to your test results.
  • Do more Reading tests.

More Tests

    All content of site and practice tests © 2022 Jack.
    Quick View

    PSAT Practice Tests

    More Information